Download as docx, pdf, or txt
Download as docx, pdf, or txt
You are on page 1of 31

How many numbers that are not divisible by 6 divide evenly into 264,600?

(A) 9
(B) 36
(C) 51
(D) 63
(E) 72

http://gmatclub.com/forum/how-many-numbers-that-are-not-divisible-by-6-divide-evenly-
126647.html

If a natural number p has 8 factors, then which of the following cannot be the difference between
the number of factors of p^3 and p?

A. 14
B. 30
C. 32
D. 56
E. None of these

http://gmatclub.com/forum/if-a-natural-number-p-has-8-factors-then-which-of-
the-142265.html

The next set of PS questions. I'll post OA's with detailed explanations after some discussion.
Please, post your solutions along with the answers.

1. The length of the diagonal of square S, as well as the lengths of the diagonals of rhombus R
are integers. The ratio of the lengths of the diagonals is 15:11:9, respectively. Which of the
following could be the difference between the area of square S and the area of rhombus R?

I. 63
II. 126
III. 252

A. I only
B. II only
C. III only
D. I and III only
E. I, II and III

Solution: fresh-meat-151046-80.html#p1215318

2. Set S contains 7 different letters. How many subsets of set S, including an empty set, contain
at most 3 letters?

A. 29
B. 56
C. 57
D. 63
E. 64

Solution: fresh-meat-151046-100.html#p1215323
3. How many different subsets of the set {0, 1, 2, 3, 4, 5} do not contain 0?

A. 16
B. 27
C. 31
D. 32
E. 64

Solution: fresh-meat-151046-100.html#p1215329

4. The functions f and g are defined for all the positive integers n by the following rule: f(n) is
the number of perfect squares less than n and g(n) is the number of primes numbers less than
n. If f(x) + g(x) = 16, then x is in the range:

A. 30 < x < 36
B. 30 < x < 37
C. 31 < x < 37
D. 31 < x < 38
E. 32 < x < 38

Solution: fresh-meat-151046-100.html#p1215335

5. Which of the following is a factor of 18!+1?

A. 15
B. 17
C. 19
D. 33
E. 39

Solution: fresh-meat-151046-100.html#p1215338

6. If the least common multiple of a positive integer x, 4^3 and 6^5 is 6^6. Then x can take
how many values?

A. 1
B. 6
C. 7
D. 30
E. 36

Solution: fresh-meat-151046-100.html#p1215345

7. The greatest common divisor of two positive integers is 25. If the sum of the integers is 350,
then how many such pairs are possible?

A. 1
B. 2
C. 3
D. 4
E. 5

Solution: fresh-meat-151046-100.html#p1215349

8. The product of a positive integer x and 377,910 is divisible by 3,300, then the least value of
x is:

A. 10
B. 11
C. 55
D. 110
E. 330

Solution: fresh-meat-151046-100.html#p1215359

9. What is the 101st digit after the decimal point in the decimal representation of 1/3 + 1/9 +
1/27 + 1/37?

A. 0
B. 1
C. 5
D. 7
E. 8

Solution: fresh-meat-151046-100.html#p1215367

10. If x is not equal to 0 and x^y=1, then which of the following must be true?

I. x=1
II. x=1 and y=0
III. x=1 or y=0

A. I only
B. II only
C. III only
D. I and III only
E. None

Solution: fresh-meat-151046-100.html#p1215370

If 10! - 2*(5!)^2 is divisible by 10^n, what is the greatest value of n?

A. 1
B. 2
C. 3
D. 4
E. 5

The units digit of (35)^(87) + (93)^(46) is:


(A) 2
(B) 4
(C) 6
(D) 8
(E) 0

http://gmatclub.com/forum/the-units-digit-of-35-87-93-46-is-156211.html

The next set of medium/hard PS questions. I'll post OA's with detailed explanations on Friday.
Please, post your solutions along with the answers.

1. The distance from the Y-axis to point K is 1/3 of the distance from the X-axis to point K. If
the coordinates of K are (-3, y), what is the distance between point K and X-axis?

A. 1/2
B. 1
C. 3
D. 4.5
E. 9.

Solution: new-set-of-mixed-questions-150204-100.html#p1208436

2. What is the area of a region enclosed by |x/3|+|y/9|=10?

A. 675
B. 1350
C. 2700
D. 5400
E. 10800

Solution: new-set-of-mixed-questions-150204-100.html#p1208441

3. Three workers, A, B, and C, can complete a certain task in 10, 5 and x hours respectively. A
starts working alone and 2 hours later B joins. After another 2 hours joins C. After that A, B,
and C together complete the task in 15 minutes. What is the value of x?

A. 1
B. 1.25
C. 2
D. 2.5
E. 4

Solution: new-set-of-mixed-questions-150204-100.html#p1208445

4. A draining pipe can empty a pool in 4 hours. On a rainy day, when the pool is full, the
draining pipe is opened and the pool is emptied in 6 hours. If rain inflow into the pool is 3 liters
per hour, what is the capacity of the pool?

A. 9 liters
B. 18 liters
C. 27 liters
D. 36 liters
E. 45 iters

Solution: new-set-of-mixed-questions-150204-100.html#p1208449

5. For a certain set of numbers, if x is in the set, then both -x^2 and -x^3 are also in the set. If
the number 1/2 is in the set , which of the following must also be in the set ?

I. -1/64
II. 1/64
III. 1/2^(1/3)

A. I only,
B. II only,
C. III only,
D. I and II only
E. I, II and III

Solution: new-set-of-mixed-questions-150204-100.html#p1208454

6. A team contributes total of $399 from its members. If each member contributed at least $10,
and no one contributed $19, what is the greatest number of members the club could have?

A. 37
B. 38
C. 39
D. 40
E. 41

Solution: new-set-of-mixed-questions-150204-100.html#p1208457

7. Mary spent 64 percent of her salary on food (including meat) and 16% of her salary on meat.
What percent of the salary spent on food were not spent on meat?

A. 16%
B. 25%
C. 32%
D. 48%
E. 75%

Solution: new-set-of-mixed-questions-150204-100.html#p1208459

8. Usually Holly leaves home to school at 9:00, however today she left home 20 minutes later.
In order to be at school on time she increased her usual speed by 20% and still was at school 15
minutes later than usual. What is her usual time from home to school?

A. 15 minutes
B. 20 minutes
C. 25 minutes
D. 30 minutes
E. 210 minutes

Solution: new-set-of-mixed-questions-150204-100.html#p1208462

9. If x and y are integers and x + y = -12, which of the following must be true?

A. Both x and y are negative


B. xy > 0
C. If y < 0, then x > 0
D. If y > 0, then x < 0
E. x - y > 0

Solution: new-set-of-mixed-questions-150204-100.html#p1208466

10. If n is a non-negative integer and the remainder when 3^n is divided by 4 is a multiple of 3,
then which of the following must be true?

I. n^2 divided by 4 yields the reminder of 1


II. (-2)^n is less than 0
III. n is a prime number

A. I only
B. II only
C. III only
D. I and II only
E. II and III only

Solution: new-set-of-mixed-questions-150204-100.html#p1208471

BONUS QUESTION:
11. Certain bowl contains 5 red marbles and 3 blue marbles only. One by one, every marble is
drawn at random and without replacement. What is the probability that the seventh marble
drawn is NOT blue?

A. 7/8
B. 3/4
C. 2/3
D. 5/8
E. 3/8

Solution: new-set-of-mixed-questions-150204-100.html#p1208473
http://gmatclub.com/forum/new-set-of-mixed-questions-150204.html

If N is the product of all multiples of 3 between 1 and 100, what is the greatest integer m for

which is an integer?

A. 3
B. 6
C. 7
D. 8
E. 10

http://gmatclub.com/forum/if-n-is-the-product-of-all-multiples-of-3-between-1-
and-101187.html

List L: ABC, BCA, CAB

In list L above, there are 3 positive integers, where each of A, B, and C is a different nonzero digit.
Which of the following is the sum of all the positive integers that MUST be factors of the sum of the
integers in list L?

A. 47
B. 114
C. 152
D. 161
E. 488

http://gmatclub.com/forum/in-list-l-above-there-are-3-positive-integers-where-
each-144297.html

If the number x3458623y is divisible by 88, what is the value of x?

A. 1
B. 2
C. 3
D. 4
E. 5

If n is an integer greater than 6, which of the following must be divisible by 3 ?

(A) n(n + 1)(n 4)


(B) n(n + 2)(n 1)
(C) n(n + 3)(n 5)
(D) n(n + 4)(n 2)
(E) n(n + 5)(n 6)

http://gmatclub.com/forum/if-n-is-an-integer-greater-than-6-which-of-the-
following-mu-139279.html

For how many different positive integers n is a divisor of n^3 + 8?

A. None
B. One
C. Two
D. Three
E. Four

http://gmatclub.com/forum/for-how-many-different-positive-integers-n-is-a-
divisor-of-n-103821.html

Set S consists of numbers 2, 3, 6, 48, and 164. Number K is computed by multiplying one random
number from set S by one of the first 10 non-negative integers, also selected at random. If Z=6^K,
what is the probability that 678,463 is not a multiple of Z?

A. 10%
B. 25%
C. 50%
D. 90%
E. 100%

http://gmatclub.com/forum/set-s-consists-of-numbers-2-3-6-48-and-164-number-
k-is-61878.html

How many positive integers less than 30 are either a multiple of 2, an odd prime number, of the
sum of a positive multiple of 2 and an odd prime?

A. 29
B. 28
C. 27
D. 25
E. 23

http://gmatclub.com/forum/how-many-positive-integers-less-than-30-are-either-a-
127362.html

If k is a positive integer, which of the following must be divisible by 24?

(A) (k 4)(k)(k + 3)(k + 7)


(B) (k 4)(k 2)(k + 3)(k + 5)
(C) (k 2)(k + 3)(k + 5)(k + 6)
(D) (k + 1)(k + 3)(k + 5)(k + 7)
(E) (k 3)(k + 1)(k + 4)(k + 6)

http://gmatclub.com/forum/if-k-is-a-positive-integer-which-of-the-following-must-
be-130281.html

If x, a, and b are positive integers such that when x is divided by a, the remainder is b and when x
is divided by b, the remainder is a-2, then which of the following must be true?

A. a is even
B. x+b is divisible by a
C. x-1 is divisible by a
D. b=a-1
E. a+2=b+1
http://gmatclub.com/forum/if-x-a-and-b-are-positive-integers-such-that-when-x-is-
101946.html

How many prime numbers n exist such that 90 < n < 106 and n is a factor of 99999919?

A. Zero
B. One
C. Two
D. Three
E. More than three

http://gmatclub.com/forum/how-many-prime-numbers-n-exist-such-that-90-n-
106-and-154204.html

In a certain class of students, 25% received As on the final exam and 2/3 of these students received
As in the class as a whole. If 15 students in the class received As on the final exam but did not
receive an A in the class, how many students did not receive an A on the final exam?

A. 45
B. 60
C. 105
D. 135
E. 180

http://gmatclub.com/forum/in-a-certain-class-of-students-25-received-a-s-on-the-
fina-153229.html

If an integer n is to be chosen at random from the integers 1 to 96, inclusive, what is the
probability that n(n + 1)(n + 2) will be divisible by 8?

A. 1/4
B. 3/8
C. 1/2
D. 5/8
E.

http://gmatclub.com/forum/if-an-integer-n-is-to-be-chosen-at-random-from-the-
integers-126654.html

If a and b are positive integers, and , how many different possible


values of b are there?
(A) 2
(B) 3
(C) 4
(D) 6
(E) 12

http://gmatclub.com/forum/if-a-and-b-are-positive-integers-and-147953.html

If two of the factors of the equation x^2-y^2 and x^2+4xy+4y^2 are chosen at random, what is the
probability that their product contains the term 2y^2?

A. 0
B. 1/6
C. 1/3
D. 2/3
E. 1

http://gmatclub.com/forum/if-two-of-the-factors-of-the-equation-x-2-y-2-and-x-2-
4xy-4y-131345.html

How many integers less than 1000 have no factors (other than 1) in common with 1000 ?

a. 400
b. 399
c. 410
d. 420

http://gmatclub.com/forum/how-many-integers-less-than-1000-have-no-factors-
other-than-99697.html

N is positive integer and hundredth digit of 10N is 6.If N and N+13 is multiple of 13 then N+13 tenth
digit is 7. Now what is the unit digit of N.

A.7
B.5
C.8
D.4
E.0

http://gmatclub.com/forum/n-is-positive-integer-and-hundredth-digit-of-10n-is-
131644.html

If 6^y is a factor of (10!)^2, What is the greatest possible value of y ?

A. 2
B. 4
C. 6
D. 8
E. 10

http://gmatclub.com/forum/if-6-y-is-a-factor-of-10-2-what-is-the-greatest-possible-
129353.html

k is a positive integer and 225 and 216 are both divisors of k. If k=(2^a)*(3^b)*(5^c), where a, b and
c are positive integers, what is the least possible value of a+ b+ c?

A. 4
B. 5
C. 6
D. 7
E. 8

http://gmatclub.com/forum/k-is-a-positive-integer-and-225-and-216-are-both-
divisors-of-127813.html
The connection between any two positive integers a and b is the ratio of the smallest common
multiple of a and b to the product of a and b. For instance, the smallest common multiple of 8 and
12 is 24, and the product of 8 and 12 is 96, so the connection between 8 and 12 is 24/96 = 1/4

The positive integer y is less than 20 and the connection between y and 6 is equal to 1/1. How
many possible values of y are there?

A. 7
B. 8
C. 9
D. 10
E. 11

http://gmatclub.com/forum/the-connection-between-any-two-positive-integers-a-
and-b-128360.html

The number 23A4534A01 is divisible by 11. How many values can A be substituted with?

A. 1
B. 4
C. 9
D. 10
E. Infinite

http://gmatclub.com/forum/the-number-23a4534a01-is-divisible-by-11-how-
many-values-134180.html

For how many values of k is 12^12 the least common multiple of the positive integers 6^6, 8^8 and
k?

A. 23
B. 24
C. 25
D. 26
E. 27

http://gmatclub.com/forum/for-how-many-values-of-k-is-12-12-the-least-common-
multiple-86737.html

The integers from 1 to 100 inclusive are each written on a single slip of paper and dropped into a
jar. If one slip of paper is removed at random, approximately what is the probability that the
number on it is neither even nor a multiple of 3?

A. 83%
B. 67%
C. 50%
D. 33%
E. 17%

http://gmatclub.com/forum/the-integers-from-1-to-100-inclusive-are-each-
written-on-a-15459.html
If integer k is equal to the sum of all even multiples of 15 between 295 and 615, what is the
greatest prime factor of k?

A. 5
B. 7
C. 11
D. 13
E. 17

http://gmatclub.com/forum/if-integer-k-is-equal-to-the-sum-of-all-even-multiples-
of-125961.html

When the positive integer A is divided by 5 and 7, the remainder is 3 and 4, respectively. When the
positive integer B is divided by 5 and 7, the remainder is 3 and 4, respectively. Which of the
following is a factor of A-B?

(A) 12
(B) 24
(C) 35
(D) 16
(E) 30

http://gmatclub.com/forum/when-the-positive-integer-a-is-divided-by-5-and-
125591.html

If 4 is divided into the positive integer x, it leaves a remainder of 3. If 9 is divided into x, it leaves a
remainder of 4. If y is a positive integer such that x + y is divisible by 36, what is the smallest
possible value of y?

(A) 4
(B) 5
(C) 7
(D) 33
(E) 36

http://gmatclub.com/forum/if-4-is-divided-into-the-positive-integer-x-it-leaves-a-
rem-96275.html

List L: ABC, BCA, CAB

In list L above, there are 3 positive integers, where each of A, B, and C is a different nonzero digit.
Which of the following is the sum of all the positive integers that MUST be factors of the sum of the
integers in list L?

A. 47
B. 114
C. 152
D. 161
E. 488

vhttp://gmatclub.com/forum/in-list-l-above-there-are-3-positive-integers-where-each-o-
117360.html
The number 523abc is divisible by 7,8,9. Then what is the value of a*b*c

A. 504
B. 532
C. 210
D. 180
E. 280

http://gmatclub.com/forum/the-number-523abc-is-divisible-by-7-8-9-then-what-is-
the-86434.html

How many positive integers less than 5,000 are evenly divisible by neither 15 nor 21?

A. 4,514
B. 4,475
C. 4,521
D. 4,428
E. 4,349

http://gmatclub.com/forum/how-many-positive-integers-less-than-5-000-are-
evenly-divisi-116998.html

If x is an integer, then x(x 1)(x k) must be evenly divisible by three when k is any of the following
values EXCEPT

A. -4
B. -2
C. -1
D. 2
E. 5

http://gmatclub.com/forum/if-x-is-an-integer-then-x-x-1-x-k-must-be-evenly-divi-
106310.html

How many odd, positive divisors does 540 have?

A. 6
B. 8
C. 12
D. 15
E. 24

http://gmatclub.com/forum/how-many-odd-positive-divisors-does-540-have-
106082.html

For any positive integer n, the length of n is defined as the number of prime factors whose product
is n. For example, the length of 75 is 3, since 75 = 3 * 5 * 5. How many two-digit positive integers
have length 6?

A. None
B. One
C. Two
D. Three
E. Four

http://gmatclub.com/forum/for-any-positive-integer-n-the-length-of-n-is-defined-
as-th-90320.html

How many even integers n, where 100 <= n <= 200, are divisible neither by seven nor by nine?

(A) 37
(B) 38
(C) 39
(D) 40
(E) 46

http://gmatclub.com/forum/how-many-even-integers-n-where-100-n-200-are-
divisib-103779.html

In the infinite sequence , , , ,.... = . What is ?

(A) 2,245
(B) 2,645
(C) 5,290
(D) 5,545
(E) 5,790

http://gmatclub.com/forum/in-the-infinite-sequence-a-1-a-2-a-3-a-n-a-n-
156742.html

In the sequence 1, 2, 2, , , , . The value of is how many times


the value of ?

(A) 2
(B)
(C)
(D)
(E)

http://gmatclub.com/forum/in-the-sequence-1-2-2-an-an-an-1-an-156740.html

After posting some 700+ questions, I've decided to post the problems which
are not that hard. Though each question below has a trap or trick so be careful
when solving. I'll post OA's with detailed solutions after some discussion. Good
luck.

1. There are 5 pairs of white, 3 pairs of black and 2 pairs of grey socks in a drawer. If four socks
are picked at random what is the probability of getting two socks of the same color?
A. 1/5
B. 2/5
C. 3/4
D. 4/5
E. 1
Solution: 12-easy-pieces-or-not-126366.html#p1033919

2. If x is an integer and 9<x^2<99, then what is the value of maximum possible value of x minus
minimum possible value of x?
A. 5
B. 6
C. 7
D. 18
E. 20

Solution: 12-easy-pieces-or-not-126366.html#p1033921

3. Fanny and Alexander are 360 miles apart and are traveling in a straight line toward each
other at a constant rate of 25 mph and 65 mph respectively, how far apart will they be exactly
1.5 hours before they meet?
A. 25 miles
B. 65 miles
C. 70 miles
D. 90 miles
E. 135 miles

Solution: 12-easy-pieces-or-not-126366.html#p1033924

4. If -3<x<5 and -7<y<9, which of the following represent the range of all possible values of y-
x?
A. -4<y-x<4
B. -2<y-x<4
C. -12<y-x<4
D. -12<y-x<12
E. 4<y-x<12

Solution: 12-easy-pieces-or-not-126366.html#p1033925

5. The angles in a triangle are x, 3x, and 5x degrees. If a, b and c are the lengths of the sides
opposite to angles x, 3x, and 5x respectively, then which of the following must be true?
I. c>a+b
II. c^2>a^2+b^2
III. c/a/b=10/6/2

A. I only
B. II only
C. III only
D. I and III only
E. II and III only

Solution: 12-easy-pieces-or-not-126366.html#p1033930

6. Anna has 10 marbles: 5 red, 2 blue, 2 green and 1 yellow. She wants to arrange all of them in
a row so that no two adjacent marbles are of the same color and the first and the last marbles
are of different colors. How many different arrangements are possible?
A. 30
B. 60
C. 120
D. 240
E. 480

Solution: 12-easy-pieces-or-not-126366.html#p1033932

7. After 2/9 of the numbers in a data set A were observed, it turned out that 3/4 of those
numbers were non-negative. What fraction of the remaining numbers in set A must be negative
so that the total ratio of negative numbers to non-negative numbers be 2 to 1?
A. 11/14
B. 13/18
C. 4/7
D. 3/7
E. 3/14

Solution: 12-easy-pieces-or-not-126366.html#p1033933

8. There are 15 black chips and 5 white chips in a jar. What is the least number of chips we
should pick to guarantee that we have 2 chips of the same color?
A. 3
B. 5
C. 6
D. 16
E. 19

Solution: 12-easy-pieces-or-not-126366.html#p1033935

9. Julie is putting M marbles in a row in a repeating pattern: blue, white, red, green, black,
yellow, pink. If the row begins with blue marble and ends with red marble, then which of the
following could be the value of M?
A. 22
B. 30
C. 38
D. 46
E. 54

Solution: 12-easy-pieces-or-not-126366.html#p1033936

10. If is an integer and , then what is the value of n?


A. 1
B. 2
C. 3
D. 4
E. 5

Solution: 12-easy-pieces-or-not-126366.html#p1033938

11. The numbers {1, 3, 6, 7, 7, 7} are used to form three 2-digit numbers. If the sum of these
three numbers is a prime number p, what is the largest possible value of p?
A. 97
B. 151
C. 209
D. 211
E. 219

Solution: 12-easy-pieces-or-not-126366-20.html#p1033939

12. If and , what is the least value of possible?


A. -1/100
B. -1/50
C. -1/36
D. -1/18
E. -1/6

Solution: 12-easy-pieces-or-not-126366-20.html#p1033949

http://gmatclub.com/forum/12-easy-pieces-or-not-126366.html

(A) 10^(-8)
(B) 3*10^(-8)
(C) 3*10^(-4)
(D) 2*10^(-4)
(E) 10^(-4)

http://gmatclub.com/forum/topic-144735.html

The next set of medium/hard PS algebra questions. I'll post OA's with detailed explanations
after some discussion. Please, post your solutions along with the answers.

1. If , then the sum of all possible solutions for x is:

A. -2
B. 0
C. 1
D. 3
E. 5

Solution: new-algebra-set-149349-60.html#p1200948

2. The equation x^2 + ax - b = 0 has equal roots, and one of the roots of the equation x^2 + ax
+ 15 = 0 is 3. What is the value of b?

A. -64
B. -16
C. -15
D. -1/16
E. -1/64

Solution: new-algebra-set-149349-60.html#p1200950
3. If a and b are positive numbers, such that a^2 + b^2 = m and a^2 - b^2 = n, then ab in terms
of m and n equals to:

A.

B.

C.

D.

E.

Solution: new-algebra-set-149349-60.html#p1200956

4. What is the maximum value of -3x^2 + 12x -2y^2 - 12y - 39 ?

A. -39
B. -9
C. 0
D. 9
E. 39

Solution: new-algebra-set-149349-60.html#p1200962

5. If x^2 + 2x -15 = -m, where x is an integer from -10 and 10, inclusive, what is the probability
that m is greater than zero?

A. 2/7
B. 1/3
C. 7/20
D. 2/5
E. 3/7

Solution: new-algebra-set-149349-60.html#p1200970

6. If mn does not equal to zero, and m^2n^2 + mn = 12, then m could be:

I. -4/n
II. 2/n
III. 3/n

A. I only
B. II only
C. III only
D. I and II only
E. I and III only

Solution: new-algebra-set-149349-60.html#p1200973

7. If x^4 = 29x^2 - 100, then which of the following is NOT a product of three possible values of
x?

I. -50
II. 25
III. 50

A. I only
B. II only
C. III only
D. I and II only
E. I and III only

Solution: new-algebra-set-149349-60.html#p1200975

8. If m is a negative integer and m^3 + 380 = 381m , then what is the value of m?

A. -21
B. -20
C. -19
D. -1
E. None of the above

Solution: new-algebra-set-149349-60.html#p1200980

9. If , then the best approximation of x is:

A. 0
B. 1
C. 2
D. 3
E. 4

Solution: new-algebra-set-149349-60.html#p1200982

10. If f(x) = 2x - 1 and g(x) = x^2, then what is the product of all values of n for which
f(n^2)=g(n+12) ?

A. -145
B. -24
C. 24
D. 145
E. None of the above
Solution: new-algebra-set-149349-80.html#p1200987

http://gmatclub.com/forum/new-algebra-set-149349.html

For a particular company, the profit P generated by selling Q units of a certain product is given by

the formula , where z > 0. The maximum profit is achieved when


Q=

(A) 2
(B) 4
(C) 8
(D) 16
(E) 32

http://gmatclub.com/forum/for-a-particular-company-the-profit-p-generated-by-
selling-152504.html

What is the product of all the solutions of x^2 - 4x + 6 = 3 - |x - 1| ?

(A) -8
(B) -4
(C) 2
(D) 4
(E) 8

http://gmatclub.com/forum/what-is-the-product-of-all-the-solutions-of-x-2-4x-
147152.html

John and Jane started solving a quadratic equation. John made a mistake while copying the
constant term and got the roots as 5 and 9. Jane made a mistake in the coefficient of x and she got
the roots as 12 and 4.What is the equation?

A. x^2 + 4x + 14 = 0
B. 2x^2 + 7x -24 = 0
C. x^2 -14x + 48 = 0
D. 3x^2 -17x + 52 = 0
E. 2x^2 + 4x + 14 = 0

http://gmatclub.com/forum/john-and-jane-started-solving-a-quadratic-equation-
john-mad-106597.html

If p^2 13p + 40 = q, and p is a positive integer between 1 and 10, inclusive, what is the probability
that q < 0?
A. 1/10
B. 1/5
C. 2/5
D. 3/5
E. 3/10
http://gmatclub.com/forum/if-p-2-13p-40-q-and-p-is-a-positive-integer-between-
110590.html

If x and y are positive, which of the following must be greater than ?

1.

2.

3.

(A) None
(B) 1 only
(C) 2 only
(D) 1 and 3 only
(E) 2 and 3 only

http://gmatclub.com/forum/if-x-and-y-are-positive-which-of-the-following-must-
be-85276.html

If (3^4)(5^6)(7^3) = (35^n)(x), where x and n are both positive integers, how many different
possible values of n are there?

A. 1
B. 2
C. 3
D. 4
E. 6

http://gmatclub.com/forum/if-3-4-5-6-7-3-35-n-x-where-x-and-n-are-both-posit-
147916.html

For positive integer m, the m-th heptagonal number is given by the formula (5m^2 3m)/2. For
positive integer n, the n-th triangular number is the sum of the first n positive integers. Which of
the following is true for k, the smallest triangular number that is also heptagonal?

(A) 33 k 40
(B) 41 k 48
(C) 49 k 56
(D) 57 k 64
(E) 65 k 72

http://gmatclub.com/forum/for-positive-integer-m-the-m-th-heptagonal-number-
is-given-131248.html

If x > y^2 > z^4, which of the following statements could be true?

I. x>y>z
II. z>y>x
III. x>z>y

A. I only
B. I and II only
C. 1 and III only
D. II and III only
E. I, II and II

http://gmatclub.com/forum/if-x-y-2-z-4-which-of-the-following-statements-could-
be-100465.html

For which of the following functions is g(c-d) = g(c) - g(d) for all positive numbers c and d?

A g(x) = x^3
B g(x) = x+5
C g(x) =
D g(x) = 5x
E g(x) 15/x

http://gmatclub.com/forum/for-which-of-the-following-functions-is-g-c-d-g-c-g-d-
147058.html

When divided by 5, x has a remainder of 2 and y has a remainder of 1. Which of the following could
be the remainder when x + y is divided by 10?

A. 6
B. 7
C. 8
D. 9
E. 0

http://gmatclub.com/forum/when-divided-by-5-x-has-a-remainder-of-2-and-y-has-
a-152206.html

Given that x^4 25x^2 = -144, which of the following is NOT a sum of two possible values of x?

A. -7
B. -1
C. 0
D. 3
E. 7

http://gmatclub.com/forum/given-that-x-4-25x-2-144-which-of-the-following-is-
not-108466.html

The sum of the first n positive perfect squares, where n is a positive integer, is given by the

formula , where is a constant. What is the sum of the first 15 positive perfect
squares?

(A) 1,010
(B) 1,164
(C) 1,240
(D) 1,316
(E) 1,476

http://gmatclub.com/forum/the-sum-of-the-first-n-positive-perfect-squares-where-
n-is-90497.html

In the equation above, x =

(A)

(B)
(C)

(D)

(E)

http://gmatclub.com/forum/in-the-equation-above-x-149131.html

If x, y, and k are positive numbers such that (x/(x+y))(10) + (y/(x+y))(20) = k and if x < y, which of
the following could be the value of k?

A. 10
B. 12
C. 15
D. 18
E. 30

http://gmatclub.com/forum/if-x-y-and-k-are-positive-numbers-such-that-x-x-y-
128231.html

If , then =

(A) 1
(B) 4
(C) 2 2x
(D) 4x 2
(E) 6x 1

http://gmatclub.com/forum/if-rot-3-2x-root-2x-1-then-4x-107925.html

If x is equal to the sum of the even integers from m to n, inclusive,


where m and n are positive even integers, which of the following
represents the value of x in terms of m and n?

(A) (m+n/2 )(n-m/2 + 1)

(B) 3(m + n)

(c)n^2-m^2/2
(D) 6(m + n)

(E) (m+n/2 )(n-m/2 )

Kudos please if you like my question.

http://gmatclub.com/forum/if-x-is-equal-to-the-sum-of-the-even-integers-from-m-
to-n-151436.html

Q1)A ladder of a fire truck is elevated to an angle of 60 degrees and extended to a length of 70
feet. if the base of the ladder is 7 fet above the ground, how many feet above the ground does the
ladder reach?
a) 35
b) 42
c) 35 \sqrt{3}
d) 7+ 35\sqrt{3}
e) 7+42\sqrt{3}

Q2) the price of lunch for 15 people was $207.00, including a 15% gratuity for service. What was the
average price per person EXCLUDING the gratuity?
a)11.73
b)12
c)13.8
d)14
e)15.87

Q3) for any psitive integer n, the sum fo the first n positive integers equals [n(n+1)]/2. what si the
sum of all the even integers betwen 99 and 301?
a) 10,100
b) 20,200
c) 22,650
d) 40,000
e) 45,150

Q4)Last year the price per share of stock X increased by k percent and the earnings per share of
stock X increased by m percent, where k is greater than m. By what percent is the ratio of price
per share to the earnings per share increase, in terms of k and m?
a) k/m
b) k-m
c) [100(k-m)]/ 100+k
d) [100(k-m)]/[100+m]
e) [100(k-m)/(100+k+m)

Q5) of the 300 subjects who participated in an experiment using virtual reality therapy to reduce
their fear of heights, 40% experienced sweaty palms, 30% experienced vomiting, and 75%
experienced dizziness. If all the subjects experienced at least one of these effects, and 35 % of the
subjects experienced exactly two of these effects, how may of the subjects experienced only one
of these effects?
a)105
b)125
c)130
d)180
e)195

Q6) if m>0 and x is m percent of y, then in terms of m, y is what percent of x?


a)100m
b)1/(100m)
c) 1/m
d) 10/m
e) 10,000/m

Q7) Seven pieces if rope have an average of length of 68 cmand median length of 84 cm. if the
length f the longest piece of rope is 14 cm more than 4 times the length of the shortest piece of
rope, what si maximum possible length , in cm, of the longest piece of rope?
a)82
b)118
c)120
d)134
e)152

Q8) [0.99999999/1.0001]- [0.99999991/1.0003]=


a) 10^-8
b) 3(10^-8)
c) 3 (10^-4)
d) 2(10^-4)
e) 10^-4

Q9) List T consists of 30 positive decimals, none of which is an integer, and the sum of 30 decimals
is S. the estimated sum of the 30 decimals, E is defined as follows. Each decimal in T whose tenths
digit is even is rounded up to the nearest integer, and each decimal in T whose tenths digit is odd is
rounded down to the nearest integer; E is the sum of the resulting integers. If 1/3 of the decimals
in T have a tenths digit that is even, which of the following is a possible value of E-S?
1. -16
2. 6
3.10

A) 1 only
B) 1 and 2 only
C) 1 and 3 only
D) 2 and 3 only
E) 1,2 and 3

http://gmatclub.com/forum/urgent-help-required-og-13-questions-150572.html

If 2^x + 2^y = x^2 + y^2, where x and y are nonnegative integers, what is the greatest possible
value of |x y|?

(A) 0
(B) 1
(C) 2
(D) 3
(E) 4
http://gmatclub.com/forum/if-2-x-2-y-x-2-y-2-where-x-and-y-are-nonnegative-inte-
88016.html

If x^2 + y^2 = 100, and x0, and y0, the maximum value of x + y is

A. Less than 10
B. Greater than or equal to 10 and less than 14
C. Greater than 14 and less than 19
D. Greater than 19 and less than 23
E. Greater than 23

http://gmatclub.com/forum/if-x-2-y-2-100-and-x-0-and-y-0-the-maximum-value-
of-x-147064.html

The number of straight line miles traveled downriver in one hour by Lucy's boat is approximated
within a linear range by 3n + 4, where n represents her fuel consumption in units/hr. Suppose that
traveling x miles requires k hours at a fuel rate of 7 units/hr, but that increasing her fuel
consumption by 5 units/hr would allow her to travel 40% further in 1 fewer hour. How far would she
travel in k hours at a fuel rate of 10 units/hr?

A. 8
B. 200
C. 225
D. 236
E. 272

http://gmatclub.com/forum/the-number-of-straight-line-miles-traveled-downriver-
in-one-144805.html

If the diagonal of rectangle Z is d, and the perimeter of rectangle Z is p, what is the area of
rectangle Z, in terms of d and p?

(A) (d^2 p)/3


(B) (2d^2 p)/2
(C) (p d^2)/2
(D) (12d^2 p^2)/8
(E) (p^2 4d^2)/8

http://gmatclub.com/forum/if-the-diagonal-of-rectangle-z-is-d-and-the-perimeter-
of-104205.html

If where a and b are integers,which of the following could be the value of b?

I. 1
II. 2
III. 3

(A) I only
(B) II only
(C) I and II only
(D) I and III only
(E) I, II and III only
http://gmatclub.com/forum/if-ab-2-3ab-18-a-1-a-2-0-where-a-and-b-are-integers-
144196.html

If the function Q is defined by the formula Q = 5w/(4x(z^2)) , by what factor will Q be multiplied if
w is quadrupled, x is doubled, and z is tripled?

A. 1/9
B. 2/9
C. 4/9
D. 3/9
E. 2/27

http://gmatclub.com/forum/if-the-function-q-is-defined-by-the-formula-q-5w-4x-z-
114702.html

If b, c, and n are positive numbers such that 30(2b/(c+2b)) + 15(c/(c+2b)) = n, and c < b, which of
the following could be the value of n?

A. 15
B. 25
C. 27
D. 35
E. 45

http://gmatclub.com/forum/if-b-c-and-n-are-positive-numbers-such-that-30-2b-c-
2b-105578.html

For a Fibonacci sequence, from third term onwards each term is the sum previous 2 terms. If the
difference in squares of seventh and sixth terms of this sequence is 517, what will be the tenth
term of this sequence?

A. 147
B. 76
C. 123
D. can't be determined.

http://gmatclub.com/forum/for-a-fibonacci-sequence-from-third-term-onwards-
each-term-86545.html

If f(x)=5x^2 and g(x)=x^2 + 12x + 85, what is the sum of all values for k such that f (k+2)=g(2k)?

http://gmatclub.com/forum/if-f-x-5x-2-and-g-x-x-2-12x-85-what-is-the-sum-of-all-
85989.html

In the infinite sequence A, An = X^(n-1) + X^n + X^(n+1) + X^(n+2) + X^(n+3) where x is a positive
integer constant. For what value of n is the ratio of An to x(1+x(1+x(1+x(1+x)))) equal to X^5?

A. 8
B. 7
C. 6
D. 5
E. 4
http://gmatclub.com/forum/in-the-infinite-sequence-a-an-x-n-1-x-n-x-n-1-x-
92110.html

If x and y are positive, which of the following must be greater than ?

1.

2.

3.

(A) None
(B) 1 only
(C) 2 only
(D) 1 and 3 only
(E) 2 and 3 only

http://gmatclub.com/forum/if-x-and-y-are-positive-which-of-the-following-must-
be-30684.html

If x, y and k are positive numbers such that

and if x < y,

which of the following could be the value of k?

(A) 10
(B) 12
(C) 15
(D) 18
(E) 30

http://gmatclub.com/forum/if-x-y-and-k-are-positive-numbers-such-that-
86172.html

The function f is defined for each positive three-digit integer n by , where x, y


and z are the hundreds, tens, and units digits of n, respectively. If m and v are three-digit positive
integers such that f(m)=9f(v), them m-v=?
(A) 8
(B) 9
(C) 18
(D) 20
(E) 80

http://gmatclub.com/forum/the-function-f-is-defined-for-each-positive-three-digit-
100847.html
If (y+3)(y-1)-(y-2)(y-1)=r(y-1), what is the value of y?

(1) r^2=25
(2) r=5

http://gmatclub.com/forum/if-y-3-y-1-y-2-y-1-r-y-1-what-is-the-value-of-y-
119556.html

A function V(a, b) is defined for positive integers a, b and satisfies V(a, a) = a, V(a, b) = V(b, a),
V(a, a+b) = (1 + a/b) V(a, b). The value represented by V(66, 14) is ?

(a) 364 (b) 231 (c) 455 (d) 472 (e) None of the foregoing

http://gmatclub.com/forum/funtions-113905.html

If the equation x^3 - ax^2 +bx -a = 0 has three real roots then the following id true

A. a=11
B. a not equal to 1
C. b = 1
D. b not equal to 1

http://gmatclub.com/forum/if-the-equation-x-3-ax-2-bx-a-0-has-three-real-roots-
86546.html

If the sequence , , , , , is such that = 3 and =2 1 for n 1,


then =

A.
B.
C.
D. -1
E. -1

http://gmatclub.com/forum/if-the-sequence-x-1-x-2-x-3-x-n-is-such-that-x-
85385.html

x, y, x + y, x 4y, xy, 2y

For the set of terms shown above, if y > 6 and the mean of the set equals y + 3, then the median
must be

A. (x + y)/2
B. y + 3
C. y
D. (3y)/2
E. x/3 + y

http://gmatclub.com/forum/wed-q1-mean-question-85309.html

The function f is defined for all +ve integers n by the following rule: f(n) is the number of +ve
intergers each of which is less than n and has no +ve factor in common with n other than 1. if p is
any prime number then f(p) =

a. p-1
b. p-2
c. (p+1)/2
d. (p-1)/2
e. 2

http://gmatclub.com/forum/functions-84571.html

If one root of x^2+px+12=0 is 4, and the equation x^2+px+q=0 has equal roots then the value of q is

A. 49/4
B. 4/49
C. 4
D. 1/4
E. 12

http://gmatclub.com/forum/if-one-root-of-x-2-px-12-0-is-4-and-the-equation-x-2-
px-q-106006.html

If a, b, c, d are each positive, a+b+c+d=8, a^2+b^2+c^2+d^2=25 and c=d, then what is the greatest
value of c?

(A) 1/2
(B) 3/2
(C) 5/2
(D) 7/2

http://gmatclub.com/forum/if-a-b-c-d-are-each-positive-a-b-c-d-8-a-2-b-2-c-2-d-
87705.html

If the farmer sells 75 of his chickens, his stock of feed will last for 20 more days than planned, but
if he buys 100 more chickens, he will run out of feed 15 days earlier than planned. If no chickens
are sold or bought, the farmer will be exactly on schedule. How many chickens does the farmer
have?

A. 60
B. 120
C. 240
D. 275
E. 300

http://gmatclub.com/forum/if-the-farmer-sells-75-of-his-chickens-his-stock-of-
feed-85752.html

If x, y, and k are positive numbers such that and if x < y, which of


the following could be the value of k?

A. 10
B. 12
C. 15
D. 18
E. 30

http://gmatclub.com/forum/if-x-y-and-k-are-positive-numbers-such-that-
85382.html

If x, y, and k are positive numbers such that [X/(X+Y)][10] + [Y/(X+Y)][20] = k and if x < y, which of
the following could be the value of k?

A. 10
B. 12
C. 15
D. 18
E. 30

http://gmatclub.com/forum/if-x-y-and-k-are-positive-numbers-such-that-
97411.html

You might also like